4th grade math question:

jessica bought 4 gallons of paint. Jessica needed to use 3/4 of the paint to paint her living room and dining room. How many gallons did she use, write the number of gallons.

Answers

Answer 1

Answer:

1 gallon

Step-by-step explanation:

gallons of paint needed; 4

gallons used: (3/4)*4

= 1 gallon was used

Answer 2

Answer:

Answer is 3 gallons

Step-by-step explanation:

1 gallon= 25%

2 gallons=50%

3 gallons=75%

3/4 is equivalent to 75%

Hope this helped!


Related Questions

X/30 + X/40= 1
Solve for x

Answers

Answer:

17 1/7 I think

Step-by-step explanation:

x=17.1

Step-by-step explanation:

[tex] \frac{x}{30} + \frac{x}{40} = 1 \\ \frac{3x + 4x}{120} = 1 \\ \frac{7x}{120} = 1 \\ \frac{7x}{120} = 1 \\ 7x = 120 \\ x = \frac{120}{7} \\ x =17.1 [/tex]

A standard deck of 52 cards has 4 suits (spades, clubs, hearts, and diamonds) with 13 different cards (ace, 2, 3, 4, 5, 6, 7, 8, 9, 10, jack, queen, king) in each suit. If you are dealt exactly two cards from the deck without replacement, what is the probability that you are dealt a pair (matching cards in different suits)

Answers

Answer:

P(a pair with matching cards in different suits) = 1/52

Step-by-step explanation:

We are told that there are 4 suites and each suit has 13 different cards. This is a total of 52 cards.

Thus;

Probability of selecting one card of a particular suit = 13/52 = 1/4

If we now want to select a matching card of another suit without replacing the first one, then, we now have; 52 - 13 = 39 cards. Now, there are only 3 matching cards of the 3 remaining suits that is same as the first card drawn.

Thus; probability = 3/39 = 1/13

Thus;

P(a pair with matching cards in different suits) = 1/4 × 1/13

P(a pair with matching cards in different suits) = 1/52

Find the distance between the pair of points: (2,6) and (0,−3).

Answers

Answer:

d = √85

Step-by-step explanation:

d^2 = (X2 - X1)^2 + (Y2 - Y1)^2

= (2 - 0)^2 + (6 + 3)^2

= 4 + 81

d = √85

if gh¢400.00 is deposited at a bank at 10% per annum compounded quarterly, how long will it take the deposit to double it value.​

Answers

9514 1404 393

Answer:

  7 years

Step-by-step explanation:

The multiplier each quarter is 1 +10%/4 = 1.025. The number of quarters, n, that must pass before the value is multiplied by 2 is given by ...

  1.025^n = 2

Taking logarithms, we can solve for n:

  n×log(1.025) = log(2)

  n = log(2)/log(1.025) ≈ 28.071

It will take 28 quarters, or 7 years, for the value to double.

Jake has a 1 in 5 chance of winning the egg and spoon race.
what is the probability that he will NOT win the race
8%
20%
60%
80%

Answers

Answer:

Step-by-step explanation:

1 in 5 = 1/5 = 20% that he wins

Probability that he will NOT win

= 1 - 20%

= 100% -20%

= 80%

Answer:

80%

Step-by-step explanation:

percentage of winning

1/5×100%=20%

percentage of not winning

100%_20%=80%

Cannot seem to figure this one out.

Answers

Answer:

cosB = 2√6 / 7

Step-by-step explanation:

use the pythagorean theorem to find the missing side

a² + b² = c²

a² + 5² = 7²

a² + 25 = 49

a² = 24

a = √24

a = 2√6

--------------------------

cosB = adj/hyp

cosB = 2√6 / 7

Soosie's Cookie Company had fixed costs of $1250 and variable costs of $4.25 per dozen gourmet cookies that were baked and packaged for sale. Write an equation that can be used to determine the total cost when x dozens of cookies are baked and sold. Then determine the total cost of baking and selling 85 dozen gourmet cookies.

Answers

Answer:

$1250.00 + (x)$4.25 =

Step-by-step explanation

So fixed costs are $1,250.00.....every dozen you bake add $4.25 to the costs, cook one dozen you have $1,250.00 plus $4.25 or a total of $1,254.25.

Cook 10 dozen and you add $45.25 = the $1,250.00 for a new total of $1,295.25.

Question 2.1
Solve the following problem. Round to one decimal place if
necessary. If your answer is correct you will see an image
appear on your screen.

Answers

Answer:

x = 5.3

Step-by-step explanation:

Reference angle (θ) = 24°

Opposite side to the reference angle = x

Adjacent side = 12

Apply TOA, which is:

Tan θ = Opp/Adj

Plug in the values

Tan 24° = x/12

12*Tan 24° = x

5.34274422 = x

x = 5.3 (approximated to one decimal place)

Which of the following is the product of 7/8 and -4/21?  a.- 1/6 b.1/12 c.- 16/21 d.- 147/32​

Answers

Answer:

A. -1/6

Step-by-step explanation:

7/8 × -4/217 can be divided by 21 wich will make the fraction -4/21 turn to -4/3 and the fraction 7/8 turn to 1/8-4 can be divided by eight which will make the fraction 1/8 turn to 1/2 and the fraction -4/3 will turn to -1/31/2 × -1/3 = -1/6

The product of 7/8 and -4/21 is a.- 1/6.

What is the product?

In mathematics, a product is the result of multiplication, or an expression that identifies factors to be multiplied.

Now the given numbers are,

7/8 and -4/21

now taking 7/8 and since 8= 4*2

so, 7/8 = 7/(4*2)

Again taking -4/28 and since 28 = 7*4

so, -4/21 = -4/(7*3)

hence the product of 7/8 and -4/21 is given as,

7/8*(-4/21) = 7/(4*2) x -4/(7*3)

taking alike terms together we get,

7/8*(-4/28) = (7/7)(4/4)(-1/2*3)

⇒ 7/8*(-4/28) = -1/6

Hence the correct option is a.-1/6

Therefore,The product of 7/8 and -4/21 is a.- 1/6.

Learn more about product :

https://brainly.com/question/1549083

#SPJ6

you spin the spinner once
What is P(5)?

Answers

0-10

A spinner is split into 11 different pieces so you are less likely to land on five than the other numbers.

The probability of getting 5 on spinner is 0.25.

What is the probability?

Probability can be defined as the ratio of the number of favourable outcomes to the total number of outcomes of an event.

We know that, probability of an event = Number of favourable outcomes/Total number of outcomes.

Given that, on the spinner there are four numbers {3, 4, 5, 6}.

Number of favorable outcomes = 1

Total number of outcomes = 4

So, P(5) = 1/4

= 0.25

Therefore, the probability of getting 5 on spinner is 0.25.

To learn more about the probability visit:

https://brainly.com/question/11234923.

#SPJ2

Please help me figure out which is the correct answer, i have attached the picture for you to look at. Thanks so much!

Answers

Answer:

The answer is B ;)

Step-by-step explanation:

4x+5 is greater than or equal to 13

subtract 5 from each side to get 4x is greater than or equal to 8

then divide both sides by 2 to get

x is greater than or equal to 2

SOMEONE SMART PLEASE HELP!! If the area of a triangle is 32 yd^2 and the base is 6.4 yds long, find the height​

Answers

Answer:

Height = 10 yd

Step-by-step explanation:

GIVEN :-

Area of the triangle = 32 yd²Base of the triangle = 6.4 yd

TO FIND :-

The height of the triangle

GENERAL FORMULAE TO BE USED IN THIS QUESTION :-

For a triangle with base 'b' & height 'h' , its area = [tex]\frac{1}{2} \times b \times h[/tex]

SOLUTION :-

[tex]Area = \frac{1}{2} \times Base \times Height[/tex]

[tex]=> 32 = \frac{6.4h}{2}[/tex]

[tex]=> 6.4h = 32 \times 2 = 64[/tex]

[tex]=> h = \frac{64}{6.4} = 10\: yd[/tex]

Find the volume of this figure

Answers

Answer:

A

Step-by-step explanation:

36*12*6+12*24*3=3456

Answer:

3456

Step-by-step explanation:

36*12*6 + 24*12*3

just add up the two volumes

The city of Raleigh has 10000 registered voters. There are two candidates for city council in an upcoming election: Brown and Feliz. The day before the election, a telephone poll of 200 randomly selected registered voters was conducted. 72 said they'd vote for Brown, 118 said they'd vote for Feliz, and 10 were undecided. Give the sample statistic for the proportion of voters surveyed who said they'd vote for Brown. Note: The proportion should be a fraction or decimal, not a percent.

Answers

Answer:

0.36

Step-by-step explanation:

Given, There Are 10000 Registered Voters in a City & Two Candidates Brown & Felix .

The Day Before Election , A Telephonic Poll Of 200 Randomly Registered Voters Was Conducted .

72 said they would vote for Brown , 118 Said they would vote for felix & 10 Were Undicided .

Now, The sample statistic is the number who would vote for Brown divided by the total number in the sample, or 72/200 = 0.36    

& To Find How Many of 10000 voters to vote Brown Just Multiply 0.36 With 10000 ⇔ 0.36×10000=3600

suppose a charity received a donation of $26.2 million. if this represents 49% of the charity's donated funds, what is the total amount of its donated funds? round your answer to the nearest million​

Answers

Answer:

53 million.

Step-by-step explanation:

$26.2 / total = 0.49 ($ in millions)

Then total = (26.2) ÷ 0.49 = 53.47 millions which we can round up to 53 million $

Write the product in simplest form: 1 2/3 x 3 4/5=

Answers

Answer:

19/3

Step-by-step explanation:

1 2/3 = (1*3 + 2)/3 = 5/3

3 4/5 = (5*3 + 4)/5 = (15 + 4)/5 = 19/5

5/3 * 19/5 = 19/3    The 5s cancel.

The simplest form is not obvious to me. I think the answer above is in simplest form, and I will put that below the answer above. But 6 1/3 is also possible.

Answer:

4 1/3

Step-by-step explanation:

1 [tex]\frac{2}{3}[/tex] × 3 [tex]\frac{4}{5}[/tex] =

[tex]\frac{5}{3}[/tex] × [tex]\frac{19}{5}[/tex] =

[tex]\frac{95}{15}[/tex] = 4 [tex]\frac{5}{15}[/tex] = 4 1/3

Use the Squeeze Theorem ​

Answers

Answer:

See Below.

Step-by-step explanation:

We want to use the Squeeze Theorem to show that:

[tex]\displaystyle \lim_{x \to 0}\left(x^2\sin\left(\frac{2}{x}\right)\right)=0[/tex]

Recall that according to the Squeeze Theorem, if:

[tex]\displaystyle g(x)\leq f(x) \leq h(x)[/tex]

And:

[tex]\displaystyle \lim_{x\to c}g(x) =\lim_{x\to c}h(x) = L[/tex]

Then:

[tex]\displaystyle \lim_{x\to c}f(x)=L[/tex]

Recall that the value of sine is always ≥ -1 and ≤ 1. Hence:

[tex]\displaystyle -1 \leq \sin\left(\frac{2}{x}\right) \leq 1[/tex]

We can multiply both sides by x². Since this value is always positive, we do not need to change the signs. Hence:

[tex]\displaystyle -x^2\leq x^2\sin\left(\frac{2}{x}\right)\leq x^2[/tex]

Let g = -x², h = x², and f = x²sin(2 / x). We can see that:

[tex]\displaystyle \lim_{x \to 0}g(x) = \lim_{ x \to 0}h(x) = 0[/tex]

And since g(x) ≤ f(x) ≤ h(x), we can conclude using the Squeeze Theorem that:

[tex]\displaystyle \lim_{x \to 0}f(x) = \lim_{x \to 0}x^2\sin\left(\frac{2}{x}\right)=0[/tex]

Can someone help me again with these math work?

Answers

Answer:

it look like its 4 and the 4 over 4/5 I think



Match each system of equations to the inverse of its coefficient matrix, A^-1and the matrix of its solution, X.

Answers

Answer: Hello the system of equations and other data required are missing attached below are the missing data

answer :

attached below

Step-by-step explanation:

1st system of equations ( top left ) = Image number 8

system of equation ( Image 10 ) = Image 4

system of equation ( Image 9 ) = Image 6

system of equations ( Image 7 ) = Image 2

attached below is a pictorial representation of the solution

you and your friend start biking to opposite directions from the same point. You travel 108 feet every 8 seconds. Your friend travels 63 feet every 6 seconds. a) how far apart are you and your friend after 15 minutes? b) after 20 minutes you take a 5- minutes rest, but your friend does not. How far apart are you and your friend after 40 minutes? Explain your reasoning.

Answers

Answer:

To find your distance apart, you can convert 15 minutes to seconds because that is what the rate is given in.

15 mins x 60 seconds = 900 seconds.

Step-by-step explanation:

In 900 seconds, there are 900/8=112.5 groups of 8 seconds. This means there are 112.5 groups of 108 feet.

112.5 x 108 = 12150 feet

12150 feet/5280 feet is approximately 2.3 miles.

For your friend you take 900/6 = 150 groups of 6 seconds.

150 x 63 = 9540 feet

9540 feet/5280 feet = 1.79 miles

1.79 miles + 2.3 miles = 2.09 miles

You and your friend are about 2.09 miles apart after 15 minutes.

B). If you go 2.3 miles every 15 minutes, that means you travel about 0.77 miles every 5 minutes. If you travel 20 more minutes (40-15-5) that would be 0.77x 4=3.07 miles more

3.07 + 2.3 = 5.37 miles after 40 minutes (you).

Your friend travels 1.79 in 15 minutes, so 1.79/3 = 0.6 miles every 5 minutes

0.6 x 5 (5 x 5 = 25 minutes)= 3 miles

1.79 + 3 miles= 4.79 miles (friend)

4.79 + 5.37 = 10.16 miles

You and your friend are about 10.16 miles apart after 40 minutes.

4(m + 3) = 18

write the solution to the equation ​

Answers

Answer:

1.5

Step-by-step explanation:

Let's begin by dividing each side by 4.

You get 18/4 or 4.5.

m+3=4.5

Now subtract 3 to each side.

m=1.5

Answer:

[tex]4(m + 3) = 18 \\ 2(m + 3) = 9 \\ 2m + 6 = 9 \\ 2m = 3 \\ m = \frac{3}{2} \\ m = 1.5[/tex]

Question 21
Find the volume.

Answers

Answer:

C

4712 cm³ exactly

Step-by-step explanation:

10² * pi is the area of the circle

then just multiply with the height

Answer:

C. 1500π ≈ 4710 cm³

Step-by-step explanation:

Volume = πr²h

Volume = π * 100 * 15

Volume = 1500π ≈ 4710 cm³

If my answer is incorrect, pls correct me!

If you like my answer and explanation, mark me as brainliest!

-Chetan K

Del siguiente conjunto de datos calcula lo que se te pide: 3,6,8,4,3,2,6,7,1,5,4,8,4

¿cual es la media? del problema
cual es la moda?

Answers

Answer:

Moda = 4

Media = 4

Step-by-step explanation:

primero tienes que ordenarlos

1,2,3,3,4,4,4,5,6,6,7,8,8

La moda es el elemento que más se repite entonces es el 4

La media es el elemento que esta al medio, en este caso son 13 números

13/2 = 6.5, el elemento que esta en la séptima posición  7 es la media

media= 4

NJa a Qi oaoanBakabhaiwowmsbzksoajns

Answer a, b and c. See image below

Answers

Answer:

a) 3/5 < 4/5

b) In general if two fractions have the same denominator, then whichever fraction has the numerator closer to its denominator will be the largest fraction.

c)  [tex]\frac{7}{10} > \frac{9}{15}[/tex]  or  [tex]\frac{7}{10}<\frac{9}{15}[/tex]

Step-by-step explanation:

a) 3/5 < 4/5

Flip the sign and the placement of the fraction so 3/5 is less then 4/5.

b) In general if two fractions have the same denominator, then whichever fraction has the numerator closer to its denominator will be the largest fraction.

c) We need to change the denominators to a common denominator to compare the size of the two fractions:

[tex]\frac{7}{10}[/tex] × [tex]\frac{3}{3}[/tex] = [tex]\frac{21}{30}[/tex]

[tex]\frac{9}{15}[/tex] ×  [tex]\frac{2}{2}[/tex] = [tex]\frac{18}{30}[/tex]

The common denominators of the two fractions is 30. Comparing the two fractions:

[tex]\frac{21}{30} >\frac{18}{30}[/tex]  or  [tex]\frac{18}{30}<\frac{21}{30}[/tex]

so we get:  [tex]\frac{7}{10} > \frac{9}{15}[/tex]  or  [tex]\frac{7}{10}<\frac{9}{15}[/tex]

Help me out thankssssss !!!!!!

Answers

Answer:

78/2=39°

Step-by-step explanation:

thx for the points

How many liters is 350 milliliters?

A. 3500 L
B. 3.50 L
C. 0.350 L
D. 0.035 L

Answers

Answer: C

Step-by-step explanation:

Step-by-step explanation:

1 L equals to 1000 mL

so 350 mL equals to 0.35L

The answer is C

a piece of ribbon is 5.4 meters long. What will be the length of 8 similar pieces?​

Answers

Answer:

43.2 meters

Step-by-step explanation:

Multiply the length of one piece of ribbon by 8 to determine the total length

5.4 * 8 =43.2 meters

5.4x8=43.2
answer: 43.2 meters

Inequality of y<-4+3 on graph

Answers

Answer:

[tex]y < - 1[/tex]

Step-by-step explanation:

[tex]y < - 4 + 3[/tex]

[tex]y < - 1[/tex]

Hope it is helpful...

[tex] \sf \: y < - 4 + 3 \\ \sf \: y < - 1[/tex]

[tex] \sf \: Just \: add \: - 4 \: and \: 3 \: and \: you \: \\ \sf will \: get \: the \: inequality \: in \: the \: simplest \: form.[/tex]

a rectangular field is 454 times as long as it is wide if the perimeter of the field is 540 yd what are the field dimensions ​

Answers

Answer:

WIDTH= 54yd, LENGTH= 216 yd

Step-by-step explanation:

Length can be denoted as L

Width can be denoted as W

From the question,

(L= 4W) ........... eqn(1)

But perimeter = [(2L + 2W)]........eqn(2)

Substitute eqn(1) into (2)

Perimeter= (8W + 2W) = 10W

From the question,

perimeter= 540 yd

540=10W

W=(540/10)= 54

W= 54 yd

Hence, WIDTH= 54yd

From equation (1) substitute W= 54 yd

(L= 4W)

L= (4×54)

L= 216 yd

Hence, LENGTH= 216 yd

The dimensions are WIDTH= 54yd, LENGTH= 216 yd

Someone help me please

Answers

9514 1404 393

Answer:

  $448

Step-by-step explanation:

The given formula will tell you the amount of interest earned:

  I = Prt

  I = $400·0.02·6 = $48

The account balance will be the sum of the original deposit and the interest it earns:

  $400 +48 = $448 . . . . balance after 6 years

Other Questions
3(x + 1)2 = 108Create a list of your steps need help english will give 5 star and thanks Jerry's Flowers had the following cost information related to its purchases of merchandise. Calculate the total cost of merchandise purchased using the information below: Invoice cost of merchandise purchases $100,000 Purchase discounts received $ 9,000 Cost of transportation-in (shipping) $ 500 Costs of purchase returns and allowances $ 400 Help please I need helpfind X What are Descartes' reasons for thinking that the idea of God is not a fiction which he himself has conjured up? Is his reasoning sound? Hydrogen carried in light phase A dog is leashed to a point in the center of a large yard, so the area the dog is able toexplore is circular. The leash is 25 feet long. What is the area of the region the dog is able toexplore. Use 3.14 for pi. Round to the nearest tenth if necessary. f(n) = 4n +7Is it geometric or arthmetric Which technology did Eli Whitney improve?A.the cotton ginB.the steamboatC.the steam engineD.the telegraph system can some one answer please molar mass of Beryllium What effect does the author's initial description of the twowomen in paragraphs 12-16 have on the passage?A. It heightens the internal conflict that LieutenantBlandford experiences.B. It resolves a conflict introduced in the openingparagraphs of the passage.C. It establishes the strong connection that existsbetween the two women.D. It illustrates the extent to which Lieutenant Blandfordis embarrassed about the situation. *Which of the following are all direct methods of flood controa. soil conservation, dams, artificial leveesb. dams, artificial levees, overflow channelsC. floodplains, artificial levees, damsd. dams, overflow channels, soil conservation What, for you, is an important moral truth in your culture? assume that cody used a weak magnet and the flake of cereal was not attracted to it what conclusion might he have drawn? which South African law is the competition policy based on NOT DRAWN TO SCALE. find x two diary entry's first indicate how you felt before you received academic report and the other express how you felt after receiving your report Drag each label to the correct location on the image.Here is a multiplication sentence:(1 4) 2 = 8Drag the numbers and the parentheses to create an equivalent multiplication sentence that illustrates the associative property of multiplication. Tony is building a new silo to store corn as animal feed. It will be a cylinder topped with a half-sphere, and must store 21 000 t of corn. The entire silo can be filled with corn. Tony wants to minimize the surface area of the silo to reduce materials and paint costs. He has the following information: 1 cubic m of corn has a mass of 700 kg. Building costs are $8/m2, taxes included. Paint comes in 3.8 L cans.Each can covers 40 sq m and costs $35,taxes included. Corn costs $140 per tonne ($140/t), taxesincluded.What is the total cost to build, paint, and fill a silo with the least surface area?